LSAT and Law School Admissions Forum

Get expert LSAT preparation and law school admissions advice from PowerScore Test Preparation.

 Administrator
PowerScore Staff
  • PowerScore Staff
  • Posts: 8917
  • Joined: Feb 02, 2011
|
#35625
Complete Question Explanation

Must Be True—SN. The correct answer choice is (B)

This stimulus provides a number of conditional statements. First, if the number of consumers who live downtown increases, the profits of downtown businesses will go up as well:
  • #dt consumers increase ..... :arrow: ..... dt profits increase
Next, the council member points out that if the cost of downtown living decreases, the number of consumers living downtown will surely increase:
  • cost of dt living decrease ..... :arrow: ..... # dtconsumers increase
These first two conditional statements create a conditional chain:
  • cost of dt living decrease ..... :arrow: ..... #dt consumers increase ..... :arrow: ..... dt profits increase
Finally, the council member provides, the downtown profits won’t increase unless traffic decreases. To diagram this statement, we can apply the unless formula:
The condition that follows “unless” is the necessary condition, and the other condition gets negated:
  • dt profits increase ..... :arrow: ..... traffic decrease
In other words, downtown profits do increase, we know that traffic must have decreased.

Since this stimulus is a fact set, and doesn’t draw any conclusion, the question that follows is, predictably, a Must be True question. The right answer is the one that passes the Fact Test; it will be the choice that can be confirmed by the conditional statements provided by the council member.

Answer choice (A): The conditional statements in the stimulus provide that a traffic decrease is necessary for downtown profits to increase, but a decrease in downtown traffic is sufficient to draw no inferences.

Answer choice (B): This is the correct answer choice, confirmed by the conditional chain diagrammed above. The council member established that if the cost of living downtown decreases, the number of consumers living downtown will increase, which, in turn, will lead to increased profits for downtown businesses.

Answer choice (C): Since a decrease in downtown traffic is sufficient to conclude nothing, this choice cannot be the right answer to this Must be True question.

Answer choice (D): This answer choice is wrong for the same reason as answer choices (A) and (C). Decreasing downtown traffic is necessary for downtown business profits to increase, but this traffic decrease is sufficient to conclude nothing, as is reflected in the diagram above.

Answer choice (E): If downtown business profits increase, the stimulus provides, downtown congestion must decrease, but there is no way to conclude what will happen to the number of consumers living downtown.
 kjohns1412
  • Posts: 5
  • Joined: May 25, 2013
|
#9598
I don't understand why answer choice B is correct. The last sentence of the stimulus seems to create a new requirement for the profits of downtown businesses to increase. Can we really say for sure that profits will increase if cost of living decreases when we don't know the status of the traffic congestion?

Thanks so much
 Adam Tyson
PowerScore Staff
  • PowerScore Staff
  • Posts: 5153
  • Joined: Apr 14, 2011
|
#9604
Hey there kjohns. This question will be clear IF you have a solid conditional diagram for it. Here's how that would look:

First sentence - If MCL (More Consumers Living there) -> P+ (Profits Increase)
Second sentence - If CL- (Cost of Living decreases) -> MCL (aha, I think I see a link before me!)
Third sentence, bringing in the Unless Equation - If P+ -> TC- (Traffic Congestion decreases) (and yet another link in the chain!)

Notice that while the third sentence is adding a new requirement, as you put it, that requirement ends up tracking along nicely in our conditional chain. The end result of all those statements is this:

CL- -> MCL -> P+ -> TC-

Getting that last sentence diagrammed correctly by applying the Unless Equation is probably the hardest part, and the one that most students who struggle with this question will probably find they got backwards or otherwise in error (maybe "P+ -> TC-").

Once we have it all strung together, you can see that B works nicely. Most of the wrong answers want to treat a decrease in traffic congestion (TC-) as a sufficient condition, but it's not sufficient for anything and so those can't be right.

I hope that helped!
 kjohns1412
  • Posts: 5
  • Joined: May 25, 2013
|
#9607
Thanks, it did!

Get the most out of your LSAT Prep Plus subscription.

Analyze and track your performance with our Testing and Analytics Package.